방정식을 더 왼쪽으로 정렬하는 방법은 무엇입니까?

방정식을 더 왼쪽으로 정렬하는 방법은 무엇입니까?

\나는 이것을 원한다

위 그림과 같이 계산 및 방정식 형식을 어떻게 작성할 수 있습니까?

나는 이 코드를 시도했다:

\documentclass[12pt,a4paper,fleqn, twocolumn]{article}
\usepackage[margin=1.5cm]{geometry}
\usepackage[utf8]{inputenc}
\usepackage[T2A]{fontenc}
\usepackage[russian]{babel}
\usepackage{titlesec}
\usepackage{graphicx}
\usepackage{amsmath}
\usepackage{esint}
    \begin{align}
    \begin{array}{l}
    P^{TE}_q = Re\frac{j|E_0|^2}{2\eta}\\
    \times \oiint\limits_S(A^i_qA^{i*}_q\bar{M}^{(1)}_q \times \bar{N}^{(1)*}_q + A^i_qA^{s*}_q\bar{M}^{(1)}_q \times \bar{N}^{(4)*}_q\\
    + A^s_qA^{i*}_q\bar{M}^{(4)}_q \times \bar{N}^{(1)*}_q + A^s_qA^{s*}_q\bar{M}^{(4)}_q \times \bar{N}^{(4)*}_q)\\
    \cdot \hat{r}ds
    \end{array}\\
    \begin{array}{l}
    P^{TM}_q = Re\frac{j|E_0|^2}{2\eta}\\
    \times \oiint\limits_S(B^i_qB^{i*}_q\bar{N}^{(1)}_q \times \bar{M}^{(1)*}_q + B^i_qB^{s*}_q\bar{N}^{(1)}_q \times \bar{M}^{(4)*}_q\\
    + B^s_qB^{i*}_q\bar{N}^{(4)}_q \times \bar{M}^{(1)*}_q + B^s_qB^{s*}_q\bar{M}^{(4)}_q \times \bar{M}^{(4)*}_q)\\
    \cdot \hat{r}ds
    \end{array}
    \end{align}

하지만 이것만 사세요여기에 이미지 설명을 입력하세요

답변1

이 같은? 내가 추천하는 것을 관찰하십시오~ 아니다옵션을 지정합니다 fleqn. 단일 align환경을 사용하고 array환경은 사용하지 않는 것이 좋습니다.

여기에 이미지 설명을 입력하세요

\documentclass[12pt,a4paper,twocolumn]{article} % no 'fleqn' option
\usepackage[margin=1.5cm]{geometry}
\usepackage{mathtools,esint}
\usepackage[utf8]{inputenc}
\usepackage[T2A]{fontenc}
\usepackage[russian]{babel}

\let\Re\relax 
\DeclareMathOperator{\Re}{Re} % redefine '\Re'
\DeclarePairedDelimiter{\abs}{\lvert}{\rvert}

\begin{document}
\hrule % just to illustrate width of column
\setcounter{equation}{24} % just for this example
\begingroup
\thickmuskip=0mu
\medmuskip=0mu
\thinmuskip=0mu
\small
\begin{subequations}
\begin{align}
P^{\mathrm{TE}}_q &= \Re \frac{j\abs{E_0}^2}{2\eta} \notag\\
&\quad\times \oiint\limits_S \Bigl(
     A^i_qA^{i*}_q\bar{M}^{(1)}_q \times \bar{N}^{(1)*}_q 
   + A^i_qA^{s*}_q\bar{M}^{(1)}_q \times \bar{N}^{(4)*}_q \notag\\
&\qquad
   + A^s_qA^{i*}_q\bar{M}^{(4)}_q \times \bar{N}^{(1)*}_q 
   + A^s_qA^{s*}_q\bar{M}^{(4)}_q \times \bar{N}^{(4)*}_q \Bigr) \notag\\
&\qquad \cdot \hat{r}\mkern3mu ds \\[1ex]
P^{\mathrm{TM}}_q &= \Re\frac{j\abs{E_0}^2}{2\eta} \notag\\
&\quad\times \oiint\limits_S \Bigl(
    B^i_qB^{i*}_q\bar{N}^{(1)}_q \times \bar{M}^{(1)*}_q 
  + B^i_qB^{s*}_q\bar{N}^{(1)}_q \times \bar{M}^{(4)*}_q \notag\\
&\qquad
  + B^s_qB^{i*}_q\bar{N}^{(4)}_q \times \bar{M}^{(1)*}_q 
  + B^s_qB^{s*}_q\bar{M}^{(4)}_q \times \bar{M}^{(4)*}_q\Bigr) \notag\\
&\qquad\cdot \hat{r}\mkern3mu ds
\end{align}
\end{subequations}
\endgroup
\hrule % just to illustrate width of column
\end{document}

부록: 문서에 공간이 부족하다면 실제로 각 방정식을 초기 스크린샷과 위 답변에 사용된 4행 대신 3행으로 조판하는 것이 가능합니다.

다음 솔루션 에서는 위의 답변 대신 \widebar(패키지에서 제공하는 매크로 mathabx) 을 사용합니다. \bar를 사용하면 \widebar게시한 스크린샷의 모양을 더 가깝게 복제하는 "모양"이 제공됩니다.

여기에 이미지 설명을 입력하세요

\documentclass[12pt,a4paper,twocolumn]{article} % no 'fleqn' option
\usepackage[margin=1.5cm]{geometry}
\usepackage{mathtools,mathabx,esint}
\usepackage[utf8]{inputenc}
\usepackage[T2A]{fontenc}
\usepackage[russian]{babel}

\let\Re\relax 
\DeclareMathOperator{\Re}{Re} % redefine '\Re'

\DeclarePairedDelimiter{\abs}{\lvert}{\rvert}

\begin{document}
\setcounter{equation}{24} % just for this example

\begin{subequations}
\begin{align}
&P^{\mathrm{TE}}_q = \Re \frac{j\abs{E_0}^2}{2\eta} 
\times \oiint\limits_{S} \Bigl\{
     A^i_qA^{i*}_q\widebar{M}^{(1)}_q \times \widebar{N}^{(1)*}_q \notag\\ 
&\quad+ A^i_qA^{s*}_q\widebar{M}^{(1)}_q \times \widebar{N}^{(4)*}_q 
   + A^s_qA^{i*}_q\widebar{M}^{(4)}_q \times \widebar{N}^{(1)*}_q  \notag\\
&\qquad+ A^s_qA^{s*}_q\widebar{M}^{(4)}_q \times \widebar{N}^{(4)*}_q \Bigr\}  \hat{r}\, ds \\[1ex]
&P^{\mathrm{TM}}_q = \Re\frac{j\abs{E_0}^2}{2\eta} 
\times \oiint\limits_{S} \Bigl\{
    B^i_qB^{i*}_q\widebar{N}^{(1)}_q \times \widebar{M}^{(1)*}_q \notag \\
&\quad+ B^i_qB^{s*}_q\widebar{N}^{(1)}_q \times \widebar{M}^{(4)*}_q 
  + B^s_qB^{i*}_q\widebar{N}^{(4)}_q \times \widebar{M}^{(1)*}_q \notag\\
&\qquad+ B^s_qB^{s*}_q\widebar{M}^{(4)}_q \times \widebar{M}^{(4)*}_q\Bigr\}  \hat{r}\, ds
\end{align}
\end{subequations}

\end{document}

답변2

줄은 더 많지만 길이는 더 짧은 다른 제안입니다. 두 개의 열 형식으로 수학을 조판하는 것은 항상 고통스럽습니다.

\documentclass[12pt,a4paper,fleqn,twocolumn]{article}
\usepackage[T2A]{fontenc}
\usepackage[utf8]{inputenc}
\usepackage[russian]{babel}
\usepackage[margin=1.5cm]{geometry}
\usepackage{titlesec}
\usepackage{graphicx}
\usepackage{amsmath,mathtools}
\usepackage{esint}

\let\Re\relax
\DeclareMathOperator{\Re}{Re}

\begin{document}

\begin{align}
\begin{split}
P^{TE}_q ={}& \Re\frac{j|E_0|^2}{2\eta} \\
{} \times \mathop{\smash[b]{\oiint\limits_S}}\Bigl(
  &  A^i_qA^{i*}_q\bar{M}^{(1)}_q \times \bar{N}^{(1)*}_q \\
  &+ A^i_qA^{s*}_q\bar{M}^{(1)}_q \times \bar{N}^{(4)*}_q \\
  &+ A^s_qA^{i*}_q\bar{M}^{(4)}_q \times \bar{N}^{(1)*}_q \\
  &+ A^s_qA^{s*}_q\bar{M}^{(4)}_q \times \bar{N}^{(4)*}_q \Bigr) \cdot \hat{r}\,ds
  \end{split}
\\
\begin{split}
P^{TM}_q ={}& \Re\frac{j|E_0|^2}{2\eta} \\
{} \times \mathop{\smash[b]{\oiint\limits_S}}\Bigl(
 & B^i_qB^{i*}_q\bar{N}^{(1)}_q \times \bar{M}^{(1)*}_q \\
 &+ B^i_qB^{s*}_q\bar{N}^{(1)}_q \times \bar{M}^{(4)*}_q \\
 &+ B^s_qB^{i*}_q\bar{N}^{(4)}_q \times \bar{M}^{(1)*}_q \\
 &+ B^s_qB^{s*}_q\bar{M}^{(4)}_q \times \bar{M}^{(4)*}_q \Bigr) \cdot \hat{r}\,ds
\end{split}
\end{align}

\end{document}

여기에 이미지 설명을 입력하세요

구조 \smash[b]는 큰 통합이 그 아래의 라인을 너무 많이 아래로 밀어내는 것을 방지하는 것입니다. 그것을 감싸면 \mathop올바른 수평 간격이 보장됩니다.

관련 정보